• 26.04.2024, 10:23
  • Registrieren
  • Anmelden
  • Sie sind nicht angemeldet.

 

Lieber Besucher, herzlich willkommen bei: Aqua Computer Forum. Falls dies Ihr erster Besuch auf dieser Seite ist, lesen Sie sich bitte die Hilfe durch. Dort wird Ihnen die Bedienung dieser Seite näher erläutert. Darüber hinaus sollten Sie sich registrieren, um alle Funktionen dieser Seite nutzen zu können. Benutzen Sie das Registrierungsformular, um sich zu registrieren oder informieren Sie sich ausführlich über den Registrierungsvorgang. Falls Sie sich bereits zu einem früheren Zeitpunkt registriert haben, können Sie sich hier anmelden.

Mathe Frage Transformation des Summationsindex

Samstag, 26. September 2009, 18:25

Hiho,

ich sitze hier vor einer Matheaufgabe aus der FH und weiß net so wirklich was ich machen soll, also was der erste Schritt zum Erfolg ist.

Habe auch schon nach Bsp. gegoogelt, aber irgendwie nichts (meiner meinung nach) brauchbares gefunden.

Hier mal die Aufgabe :

Führen Sie bei der Summe :





die folgende Transformation des Summationsindex durch:

("")

("")
j = 3

----

Also ich weiß jetzt nicht was ich machen soll um den Index und die obere Grenze zu bekommen.



Wenn jemand einen Rat, eine Page oder eine Buchempfehlung für soetwas hat, wäre ich sehr Dankbar.

Dieser Beitrag wurde bereits 2 mal editiert, zuletzt von »silentkiller« (27. September 2009, 18:56)

Sonntag, 27. September 2009, 00:59

Ich will jetzt nicht meckern, aber das ist quasi nicht zu erkennen.
Jedenfalls nicht wenn man das Forum im AC Classic Design anzeigen lässt.
Wäre es möglich das nochmal deutlicher zu machen?

Sonntag, 27. September 2009, 18:56

kanne man es jetzt erkennen ?

mcx

unregistriert

Sonntag, 27. September 2009, 23:31

Biste auch bei der FH Wolfenbüttel im WS ? :P

Also ich habe fast die gleiche Aufgabe und komme auch zu nichts 2/4 Antworten waren schon falsch...

Man sieht ja das anstatt l = 1 dort unten j = 1 steht, somit somit dachte ich dann das aus 2lx^2l-1 zu 2jx^2j-1 wird, naja falsch gedacht.

Andere möglichkeit wäre auch zu versuchen die -1 wegzubekommen, aber das geht ja nicht...dann müsste nähmlich n-1 stehn und das j = 1, müsste zu j = 0 werden aber das darf ja nicht sein.
Und somit bin ich auch grad am Verzweifeln wie die Aufgabe weiter gehen soll...

powerslide

unregistriert

Montag, 28. September 2009, 08:57

eure aufgaben sind komisch :P

hab sowas an der uni auch nie gemacht (oder nie bewusst) höchstens mal ein paar summanden herausgeholt, damit der index nicht bei -2 sondern bei +1 anfängt oder so.

und was ne trafo von i=1 -> j=1 bringen soll ist mir auch schleierhaft :P

bei der aufgabe von silentkiller .. fällt mir höchstens das ein:

wenn l = 1 und j =3 dann ist l = j - 2; damit ergibt sich der exponent zu 2j-5 und die obere summationsgrenze zu n -> n + 2

das mal als überlegng was mir so im morgendilirium einfällt. :P

Montag, 28. September 2009, 12:35

Ich kenne nur Summenindexverschiebung bei vollständiger Induktion, aber Transformation ist mir völlig unbekannt, das steht noch nicht mal in meinem Script oder dem dicken HöMa Schinken!?

Montag, 28. September 2009, 19:52

eure aufgaben sind komisch :P

hab sowas an der uni auch nie gemacht (oder nie bewusst) höchstens mal ein paar summanden herausgeholt, damit der index nicht bei -2 sondern bei +1 anfängt oder so.

und was ne trafo von i=1 -> j=1 bringen soll ist mir auch schleierhaft :P

bei der aufgabe von silentkiller .. fällt mir höchstens das ein:

wenn l = 1 und j =3 dann ist l = j - 2; damit ergibt sich der exponent zu 2j-5 und die obere summationsgrenze zu n -> n + 2

das mal als überlegng was mir so im morgendilirium einfällt. :P
ich würde mal sagen das ist richtig :D

Ähnliche Themen